2021 AMC 10A Solution
2021 AMC 10A Solution
Problem1
Solution 1
This
corresponds to
Solution 2
Problem2
Portia's high school has times as many students as Lara's high school. The two
high schools have a total of students. How many students does Portia's
high school have?
Solution 1
The following system of equations can be formed with representing the
number of students in Portia's high school and representing the number of
is
Solution 2 (One Variable)
Suppose Lara's high school has students. It follows that Portia's high school
answer is
Solution 3 (Arithmetics)
Clearly, students is times as many students as Lara's high school.
Therefore, Lara's high school has students, and Portia's
with
Problem3
The sum of two natural numbers is . One of the two numbers is divisible
by . If the units digit of that number is erased, the other number is obtained.
What is the difference of these two numbers?
Solution 1
The units digit of a multiple of will always be . We add a whenever we
multiply by . So, removing the units digit is equal to dividing by .
Let the smaller number (the one we get after removing the units digit) be . This
means the bigger number would be .
is .
is
Problem4
A cart rolls down a hill, travelling inches the first second and accelerating so
that during each successive -second time interval, it travels inches more than
during the previous -second interval. The cart takes seconds to reach the
bottom of the hill. How far, in inches, does it travel?
Recall that to find the sum of an arithmetic series, we take the average of the first
and last terms, then multiply by the number of terms,
namely
Solution 2 (Answer Choices and Modular Arithmetic)
Problem5
The quiz scores of a class with students have a mean of . The mean
of a collection of of these quiz scores is . What is the mean of the
remaining quiz scores of terms of ?
Solution 1 (Generalized)
The total score in the class is The total score on the quizzes
is Therefore, for the remaining quizzes ( of them),
~MRENTHUSIASM
Problem6
Chantal and Jean start hiking from a trailhead toward a fire tower. Jean is
wearing a heavy backpack and walks slower. Chantal starts walking at miles
per hour. Halfway to the tower, the trail becomes really steep, and Chantal slows
down to miles per hour. After reaching the tower, she immediately turns around
and descends the steep part of the trail at miles per hour. She meets Jean at
the halfway point. What was Jean's average speed, in miles per hour, until they
meet?
hours. Jean also has traveled for hours, and he travels for miles. So, his
Problem7
Tom has a collection of snakes, of which are purple and of which are
happy. He observes that all of his happy snakes can add, none of his purple
snakes can subtract, and all of his snakes that can't subtract also can't add.
Which of these conclusions can be drawn about Tom's snakes?
Solution 1
We know that purple snakes cannot subtract, thus they cannot add either. Since
happy snakes must be able to add, the purple snakes cannot be happy.
Therefore, we know that the happy snakes are not purple and the answer
is .
notation and just multiplied times . Later he found that his answer
Solution
that so .
Problem9
Solution 1
Expanding, we get that the expression
is or
to and
are :
global minimum since it's the only minimum, meaning is the minimum
Problem10
Which of the following is equivalent
to
Solution 1
All you need to do is multiply the entire equation by . Then all the terms
will easily simplify by difference of squares and you will
If you weren't able to come up with the insight, then you could just
Solution 3
After expanding the first few terms, the result after each term appears to
be
, the answer is .
-SmileKat32
click . ~rocketsri
Problem11
For which of the following integers is the base-
Solution
We
have
congruent to modulo is
Problem12
Two right circular cones with vertices facing down as shown in the figure below
contains the same amount of liquid. The radii of the tops of the liquid surfaces
are cm and cm. Into each cone is dropped a spherical marble of radius cm,
which sinks to the bottom and is completely submerged without spilling any liquid.
What is the ratio of the rise of the liquid level in the narrow cone to the rise of the
liquid level in the wide cone?
Solution 1 (Use Tables to Organize Information)
Initial Scenario
By
similar triangles:
For the narrow cone, the ratio of base radius to height is which remains
constant.
For the wide cone, the ratio of base radius to height is which remains
constant.
to
Final Scenario (Two solutions follow from here.)
Solution 1.1 (Fraction Trick)
Let the base radii of the narrow cone and the wide cone
to or
is
PS:
1. This problem uses the following fraction trick:
For unequal positive
Quick Proof
Therefore,
2. The work above shows that, regardless of the shape or the volume of the solid
dropped in, as long as the solid sinks to the bottom and is completely submerged
without spilling any liquid, the answer will remain unchanged.
~MRENTHUSIASM
Solution 1.2 (Bash)
Let the base radii of the narrow cone and the wide cone
be and respectively.
Let the rises of the liquid levels of the narrow cone and the wide cone
have
Now, we set up an equation for the volume of the narrow cone and solve
for
Next, we set up an equation for the volume of the wide cone
requested ratio is
~MRENTHUSIASM
lengths , , , ,
, and ?
Solution
Drawing the tetrahedron out and testing side lengths, we realize that the triangles
ABD and ABC are right triangles. It is now easy to calculate the volume of the
we have an answer of .
Problem14
All the roots of the
polynomial are
positive integers, possibly repeated. What is the value of ?
Solution 1:
By Vieta's formulae, the sum of the 6 roots is 10 and the product of the 6 roots is
16. By inspection, we see the roots are 1, 1, 2, 2, 2, and 2, so the function
is
. Therefore, . ~JHawk0224
Solution 2:
Using the same method as Solution 1, we find that the roots
are and . Note that is the negation of the 3rd symmetric sum
of the roots. Using casework on the number of 1's in each of
the products we
obtain
Problem15
choices )
Solution 1 (Intuition):
Visualizing the two curves, we realize they are both parabolas with the same axis
of symmetry. Now assume that the first equation is above the second, since order
doesn't matter. Then and . Therefore the number of ways to
Solution 2 (Algebra):
Setting , we find
that ,
get
Problem16
In the following list of numbers, the integer appears times in the list
for .
What is the
median of the numbers in this list?
Solution 1
There
are number
s in total. Let the median be . We want to find the median such
that or Note
as gives ,
so is the nd and rd numbers, and hence, our desired
answer. .
Solution 2
it is answer ~Lopkiloinm
from to is which is
approximately as well. Therefore, we can be relatively sure the answer
choice is
Problem17
Trapezoid has ,
Diagram
Solution 1
Angle chasing reveals that ,
that
Since by AA, we
have
have
In we have
and Finally,
Solution 3 (short)
of Let so
(1) so we get or
Thus, or And
is
of ; .
Problem18
Let be a function defined on the set of positive rational numbers with the
numbers is ?
Solution 1
so or .
Solution 2
gives Also
In we have .
In we have .
In we have .
In we
have .
In we have .
~JHawk0224 ~awesomediabrine
Solution 3 (Deeper)
have .
that . And . So
if , .
We simply need this to be greater than what we have for . Notice that for
Result Since positive powers are just repeated multiplication of the base,
have Therefore, we
have It follows
have
Problem19
The area of the region bounded by the graph
Solution 1
In order to attack this problem, we need to consider casework:
Case 1:
at .
Case 2:
at .
Case 3:
Doing the same process as before, we have ,
at .
Case 4:
at .
After combining all the cases and drawing them on the Cartesian Plane, this is
what the diagram looks like:
is which is
Problem20
Solution 1 (Bashing)
We write out the cases. These cases are the ones that
work:
Solution 2 (Casework)
Reading the terms from left to right, we have two cases:
For note that for the second and fourth terms, one of which must be
is
us .
Solution 4: Symmetry
We only need to find the # of rearrangements when 5 is the 4th digit and 5th digit.
Find the total, and multiply by 2. Then we can get the answer by adding the case
when 5 is the third digit.
in ways.
Problem21
Let be an equiangular hexagon. The
Problem22
Hiram's algebra notes are pages long and are printed on sheets of paper;
the first sheet contains pages and , the second sheet contains
pages and , and so on. One day he leaves his notes on the table before
leaving for lunch, and his roommate decides to borrow some pages from the
middle of the notes. When Hiram comes back, he discovers that his roommate
has taken a consecutive set of sheets from the notes and that the average
(mean) of the page numbers on all remaining sheets is exactly . How many
sheets were borrowed?
Solution
Suppose the roommate took pages through , or equivalently, page
is
Suppose the smallest page number removed is and pages are removed. It
Remarks:
1. pages are removed means that sheets are removed, from which must
be even.
2. must be odd, as the smallest page number removed is on the right side
(odd-numbered).
3.
of are
Since is even, we only have a few cases to consider:
If then the note pages will run out if we take pages starting from
page
So, the only possibility is from which pages are taken out,
which is sheets.
Solution 3
Let be the number of sheets borrowed, with an average page
that and .
Problem23
Frieda the frog begins a sequence of hops on a grid of squares, moving
one square on each hop and choosing at random the direction of each hop-up,
down, left, or right. She does not hop diagonally. When the direction of a hop
would take Frieda off the grid, she "wraps around" and jumps to the opposite
edge. For example if Frieda begins in the center square and makes two hops
"up", the first hop would place her in the top row middle square, and the second
hop would cause Frieda to jump to the opposite edge, landing in the bottom row
middle square. Suppose Frieda starts from the center square, makes at most four
hops at random, and stops hopping if she lands on a corner square. What is the
probability that she reaches a corner square on one of the four hops?
Solution 1 (complementary counting)
We will use complementary counting. First, the frog can go left with probability .
We observe symmetry, so our final answer will be multiplied by 4 for the 4
From the left, she either goes left to another edge ( ) or back to the center ( ).
Time for some casework.
She goes back to the center.
Now, she can go in any 4 directions, and then has 2 options from that edge. This
Subcase 1: She goes back to the left edge. She now has 2 places to go, giving
She goes back to the center in Case 1 with probability , and to the right edge
with probability
get . ~ firebolt360
have , so
Numerator (Casework)
Suppose Frieda makes hops without stopping. We perform casework on which
hop reaches a corner for the first time.
No matter which direction the first hop takes, the second hop must "wrap
around".
Hop
Two sub-cases:
The second hop "wraps around". It follows that the third hop also "wraps
around".
The numerator is
Probability
Solution 4
moves, be the probability that Frieda is on one of the four squares on the
middle of the edges after n moves, and (V for vertex) be the probability that
Frieda is on a corner after n moves. The only way to reach the center is by
moving in specific direction out of total directions from the middle of an edge,
Solution 5
Imagine an infinite grid of by squares such that there is a by square
It is easy to see that the problem is equivalent to Frieda moving left, right, up, or
When Frieda only moves horizontally or vertically for her four moves, she can do
Once Frieda reaches one of the points listed above (by using three moves), she
has four choices for her last move. Thus, a total of paths for this
case.
Our total number of paths never reaching coroners is
Solution 6 (Casework)
We take cases on the number of hops needed to reach a corner. For simplicity,
denote as a move that takes Frieda to an edge, as wrap-around move
and as a corner move. Also, denote as a move that takes us to the center.
2 Hops
Then, Frieda will have to as her set of moves. There are ways to
move to an edge, and corners to move to, for a total of cases
here. Then, there are choices for each move, for a probability
of .
3 Hops
In this case, Frieda must wrap-around. There's only one possible combination,
probability of .
4 Hops
Lastly, there are two cases we must consider here. The first case
of .
Solution 7
I denote 3x3 grid by
- HOME square (x1)
- CORN squares (x4)
- SIDE squares (x4)
Transitions:
- HOME always move to SIDE
- CORN is DONE
Problem24
The interior of a quadrilateral is bounded by the graphs
Diagram
Graph in Desmos: https://www.desmos.com/calculator/nagjmnkywx
~MRENTHUSIASM
Solution 1
The
and or and
. The slopes here are perpendicular, so the quadrilateral is a
rectangle. Plug in and graph it. We quickly see that the area
get which is near our answer, so we leave it. Testing we get , way
Solution 2 (Casework)
slope
solutions
Formula, the length and width of the rectangle are and The area
we seek is
The answer is
~MRENTHUSIASM
on consecutive vertices
The area formula
is
Solution 3 (Geometry)
Similar to Solution 2, we will use the equations of the four cases:
slope
and slope
(Note: slope )
-fnothing4994
obtain
Problem25
How many ways are there to place indistinguishable red
chips, indistinguishable blue chips, and indistinguishable green chips in the
squares of a grid so that no two chips of the same color are directly
adjacent to each other, either vertically or horizontally?
Solution 1
Call the different colors A,B,C. There are ways to rearrange these
colors to these three letters, so must be multiplied after the letters are permuted
are ways to arrange A,B, and C in the grid, and there are 6 ways
to rearrange the colors. Therefore, there are ways in total, which
is .
-happykeeper
Solution 2 (Casework)
Without the loss of generality, we place a red ball in the top-left square. There are
two cases:
Case (1): The two balls adjacent to the top-left red ball have different
Case (2): The two balls adjacent to the top-left red ball have the same
color. Each placement has permutations, as there
Case (2): All of the rows have two balls that are the same color and one that is
different. There are obviously possible configurations for the first row, for the
second, and for the third. .